Quartic Sins!

Geometry Level 3

sin 4 ( 1 0 ) + sin 4 ( 5 0 ) + sin 4 ( 7 0 ) = ? \large \sin^{4}(10^{\circ})+\sin^{4}(50^{\circ})+\sin^{4}(70^{\circ})= \ ?

Image Credit: Flickr Internet Archive Book Images .


The answer is 1.125.

This section requires Javascript.
You are seeing this because something didn't load right. We suggest you, (a) try refreshing the page, (b) enabling javascript if it is disabled on your browser and, finally, (c) loading the non-javascript version of this page . We're sorry about the hassle.

3 solutions

Pi Han Goh
Jul 10, 2015

Note that 50 = 60 10 50 = 60 - 10 and 70 = 60 10 70 = 60 - 10 . Apply the compound angle formula: sin ( A ± B ) = sin ( A ) cos ( B ) ± cos ( A ) sin ( B ) \sin(A\pm B) = \sin(A) \cos(B) \pm \cos(A) \sin(B) :

sin ( 5 0 ) = 3 2 cos ( 1 0 ) + 1 2 sin ( 1 0 ) sin ( 7 0 ) = 3 2 cos ( 1 0 ) 1 2 sin ( 1 0 ) \sin(50^\circ) = \frac{ \sqrt3}2 \cos(10^\circ) + \frac12 \sin(10^\circ) \\ \sin(70^\circ) = \frac{ \sqrt3}2 \cos(10^\circ) - \frac12 \sin(10^\circ)

By binomial expansion

sin 4 ( 5 0 ) + sin 4 ( 7 0 ) = 1 8 [ 9 cos 4 ( 1 0 ) + 18 cos 2 ( 1 0 ) sin 2 ( 1 0 ) + sin 4 ( 1 0 ) ] sin 4 ( 1 0 ) + sin 4 ( 5 0 ) + sin 4 ( 7 0 ) = 1 8 [ 9 cos 4 ( 1 0 ) + 18 cos 2 ( 1 0 ) sin 2 ( 1 0 ) + 9 sin 4 ( 1 0 ) ] = 9 8 ( cos 2 ( 1 0 ) + sin 2 ( 1 0 ) ) 2 = 9 8 \begin{aligned} \sin^4(50^\circ) + \sin^4(70^\circ) &=& \frac18 [9\cos^4 (10^\circ) + 18\cos^2(10^\circ)\sin^2 (10^\circ) + \sin^4 (10^\circ) ] \\ \sin^4(10^\circ) + \sin^4(50^\circ) + \sin^4(70^\circ) &=& \frac18 [9\cos^4 (10^\circ) + 18\cos^2(10^\circ)\sin^2 (10^\circ) + 9\sin^4 (10^\circ)] \\ &=& \frac98 (\cos^2(10^\circ) + \sin^2(10^\circ) )^2 = \boxed{\frac98} \end{aligned}

Moderator note:

Nice observation!

Chew-Seong Cheong
Jul 10, 2015

sin 4 1 0 + sin 4 5 0 + sin 4 7 0 = ( 1 cos 2 0 2 ) 2 + ( 1 cos 10 0 2 ) 2 + ( 1 cos 14 0 2 ) 2 = 1 4 [ ( 1 + cos 16 0 ) 2 + ( 1 + cos 8 0 ) 2 + ( 1 + cos 4 0 ) 2 ] = 1 4 [ 1 + 2 cos 4 0 + cos 2 4 0 + 1 + 2 cos 8 0 + cos 2 8 0 + 1 + 2 cos 16 0 + cos 2 16 0 ] = 1 4 [ 3 + 2 ( cos 4 0 + cos 8 0 + cos 12 0 cos 12 0 + cos 16 0 ) + 1 2 ( cos 8 0 + 1 + cos 16 0 + 1 + cos 32 0 + 1 ) ] = 1 4 [ 3 1 2 + 1 2 + 1 2 ( 3 + cos 4 0 + cos 8 0 + cos 12 0 cos 12 0 cos 16 0 ) ] = 1 4 [ 3 + 1 2 ( 3 1 2 + 1 2 ) ] = 1 4 [ 3 + 3 2 ] = 9 8 = 1.125 \sin^4{10^\circ} + \sin^4{50^\circ} + \sin^4{70^\circ} \\ = \left( \dfrac{1 - \cos{20^\circ}}{2} \right)^2 + \left( \dfrac{1 - \cos{100^\circ}}{2} \right)^2 + \left( \dfrac{1 - \cos{140^\circ}}{2} \right)^2 \\ = \frac{1}{4} [(1 + \cos{160^\circ} )^2 + (1 + \cos{80^\circ} )^2 +(1 + \cos{40^\circ})^2] \\ = \frac{1}{4} [1 + 2\cos{40^\circ} + \color{#D61F06}{\cos^2{40^ \circ}} + 1 + 2\cos{80^\circ} + \color{#D61F06}{\cos^2{80^\circ}} \\ \quad \quad + 1 + 2\cos{160^\circ} + \color{#D61F06}{\cos^2{160^\circ}}] \\ = \frac{1}{4} \left[3 + 2(\color{#3D99F6}{\cos{40^\circ} + \cos{80^\circ} + \cos{120^\circ}} - \cos{120^\circ} + \color{#3D99F6}{\cos{160^\circ}} ) \\ \quad \quad +\color{#D61F06}{ \frac{1}{2}(\cos{80^\circ} + 1 + \cos{160^\circ} + 1 + \cos{320^\circ} + 1)} \right] \\ = \frac{1}{4} \left[3 \color{#3D99F6}{ - \frac{1}{2}} + \frac{1}{2} + \frac{1}{2}(3 + \color{#D61F06}{\cos{40^\circ}} + \cos{80^\circ} + \cos{120^\circ} - \cos{120^\circ} \cos{160^\circ}) \right] \\ = \frac{1}{4} \left[3 + \frac{1}{2} \left(3 - \frac{1}{2} + \frac{1}{2} \right) \right] = \frac{1}{4} \left[3 + \frac{3}{2} \right] = \frac{9}{8} = \boxed{1.125}

Moderator note:

That's amazing.

Is there any other reasons / ways to get at the answer?

Jessica Wang
Jul 10, 2015

s i n 4 1 0 + s i n 4 5 0 + s i n 4 7 0 sin^{4}10^{\circ}+sin^{4}50^{\circ}+sin^{4}70^{\circ}

= ( s i n 2 1 0 ) 2 + ( s i n 2 5 0 ) 2 + ( s i n 2 7 0 ) 2 =(sin^{2}10^{\circ})^{2}+(sin^{2}50^{\circ})^{2}+(sin^{2}70^{\circ})^{2}

= ( 1 c o s 2 0 2 ) 2 + ( 1 c o s 10 0 2 ) 2 + ( 1 c o s 14 0 2 ) 2 =(\frac{1-cos20^{\circ}}{2})^{2}+(\frac{1-cos100^{\circ}}{2})^{2}+(\frac{1-cos140^{\circ}}{2})^{2}

= 1 4 ( 1 2 c o s 2 0 + c o s 2 2 0 + 1 2 c o s 10 0 + c o s 2 10 0 + 1 2 c o s 14 0 + c o s 2 14 0 ) =\frac{1}{4}\cdot \left ( 1-2cos20^{\circ}+cos^{2}20^{\circ}+1-2cos100^{\circ}+cos^{2}100^{\circ}+1-2cos140^{\circ}+cos^{2}140^{\circ} \right )

= 1 4 [ 3 2 ( c o s 2 0 + c o s 10 0 + c o s 14 0 ) + ( c o s 2 2 0 + c o s 2 10 0 + c o s 2 14 0 ) ] =\frac{1}{4}\cdot \left [ 3-2\left ( cos20^{\circ}+cos100^{\circ}+cos140^{\circ} \right )+\left ( cos^{2}20^{\circ}+cos^{2}100^{\circ}+cos^{2}140^{\circ} \right ) \right ]

c o s 2 0 + c o s 10 0 + c o s 14 0 \because \: \: cos20^{\circ}+cos100^{\circ}+cos140^{\circ}

= c o s ( 6 0 4 0 ) + c o s ( 6 0 + 4 0 ) + c o s 14 0 \: \: \: \: =cos(60^{\circ}-40^{\circ})+cos(60^{\circ}+40^{\circ})+cos140^{\circ}

2 c o s 6 0 c o s 4 0 + c o s 14 0 = c o s 4 0 + c o s 14 0 = 0 \: \: \: \: 2\: cos60^{\circ}cos40^{\circ}+cos140^{\circ}=cos40^{\circ}+cos140^{\circ}=0

Also c o s 2 2 0 + c o s 2 10 0 + c o s 2 14 0 \because \: \: cos^{2}20^{\circ}+cos^{2}100^{\circ}+cos^{2}140^{\circ}

= c o s 4 0 + 1 2 + c o s 200 + 1 2 + c o s 28 0 + 1 2 \: \: \: \: =\frac{cos40^{\circ}+1}{2}+\frac{cos200+1^{\circ}}{2}+\frac{cos280^{\circ}+1}{2}

= 1 2 ( 3 + c o s 4 0 + c o s 20 0 + c o s 28 0 ) \: \: \: \: =\frac{1}{2}\left ( 3+cos40^{\circ}+cos200^{\circ}+cos280^{\circ} \right ) ,

and c o s 4 0 + c o s 20 0 + c o s 28 0 cos40^{\circ}+cos200^{\circ}+cos280^{\circ}

= c o s ( 12 0 8 0 ) + c o s ( 12 0 + 8 0 ) + c o s 28 0 \: \: =cos(120^{\circ}-80^{\circ})+cos(120^{\circ}+80^{\circ})+cos280^{\circ}

= 2 c o s 12 0 c o s 8 0 + c o s 28 0 = c o s 8 0 + c o s 28 0 = 0 \: \: =2 \: cos120^{\circ}cos80^{\circ}+cos280^{\circ}=-cos80^{\circ}+cos280^{\circ}=0

c o s 2 2 0 + c o s 2 10 0 + c o s 2 14 0 = 1 2 ( 3 + c o s 4 0 + c o s 20 0 + c o s 28 0 ) \therefore cos^{2}20^{\circ}+cos^{2}100^{\circ}+cos^{2}140^{\circ}=\frac{1}{2}\left ( 3+cos40^{\circ}+cos200^{\circ}+cos280^{\circ} \right )

= 3 2 =\frac{3}{2}

s i n 4 1 0 + s i n 4 5 0 + s i n 4 7 0 = 1 4 ( 3 + 3 2 ) = 1.125 \therefore sin^{4}10^{\circ}+sin^{4}50^{\circ}+sin^{4}70^{\circ}=\frac{1}{4}\cdot (3+\frac{3}{2})= \boxed{1.125}

Moderator note:

There is a slightly easier way to to show that cos 2 0 + cos 14 0 + c o s 26 0 = 0 \cos 20^\circ + \cos 140^\circ + cos 260^ \circ = 0 .

Hint for first approach: Think about the real part of e i 2 0 + e i 14 0 + e i 26 0 e ^ { i 20 ^ \circ } + e^{ i 140 ^ \circ } + e ^ { i 260 ^ \circ } .

Hint for second approach: What is cos 3 θ \cos 3 \theta ?

Similarly for cos 4 0 + cos 16 0 + cos 28 0 \cos 40^\circ + \cos 160^\circ + \cos 280^ \circ .

0 pending reports

×

Problem Loading...

Note Loading...

Set Loading...